Putnam 2017/B2

Συντονιστές: cretanman, silouan, rek2

Άβαταρ μέλους
Demetres
Γενικός Συντονιστής
Δημοσιεύσεις: 8989
Εγγραφή: Δευ Ιαν 19, 2009 5:16 pm
Τοποθεσία: Λεμεσός/Πύλα
Επικοινωνία:

Putnam 2017/B2

#1

Μη αναγνωσμένη δημοσίευση από Demetres » Παρ Δεκ 08, 2017 2:51 pm

Δίνεται θετικός ακέραιος N ο οποίος μπορεί να γραφτεί ως άθροισμα k διαφορετικών θετικών ακεραίων

\displaystyle N=a+(a+1)+(a+2)+\cdots+(a+k-1)

για k=2017 αλλά όχι για οποιαδήποτε άλλη τιμή k>1. Από όλους τους θετικούς ακέραιους N με αυτήν την ιδιότητα, ποιος είναι ο μικρότερος θετικός ακέραιος a που εμφανίζεται σε αυτές τις σχέσεις;



Λέξεις Κλειδιά:
Άβαταρ μέλους
Ορέστης Λιγνός
Δημοσιεύσεις: 1835
Εγγραφή: Κυρ Μάιος 08, 2016 7:19 pm
Τοποθεσία: Χαλάνδρι Αττικής
Επικοινωνία:

Re: Putnam 2017/B2

#2

Μη αναγνωσμένη δημοσίευση από Ορέστης Λιγνός » Δευ Μάιος 28, 2018 7:54 pm

Έμεινε πολύ καιρό άλυτη ...

Αν μπορεί ο Δημήτρης, να κοιτάξει αν είναι σωστή.

Είναι N=ak+\dfrac{k(k-1)}{2}.

Θα δείξουμε ότι \textnormal{ \en min} \, a=16.

Πρώτα, ας δείξουμε ότι γίνεται το a να πάρει αυτήν την τιμή.

Έστω λοιπόν a=16, οπότε N=16 \cdot 2017+2017 \cdot 1008=2017 \cdot 2^{10}.

Έστω, πως υπάρχουν b,t \in \mathbb{N}, t \neq 2017, t \neq 1 ώστε N=b+(b+1)+ \cdots +(b+t-1).

Τότε, b+(b+1)+\cdots+(b+t-1)=2017 \cdot 2^10 \Rightarrow bt+\dfrac{t(t-1)}{2}=2017 \cdot 2^{10} \Rightarrow t(2b+t-1)=2017 \cdot 2^{11}

α) Αν t άρτιος, οπότε 2b+t-1 \equiv 1 \pmod 2, είναι t=2^{11} ή t=2017 \cdot 2^{11} (αφού ο 2017 είναι πρώτος). Και οι δύο περιπτώσεις δίνουν εύκολα άτοπο.

β) Αν t περιττός, είναι 2b+t-1 \equiv 0 \pmod 2, άρα t=1 (t \neq 2017). Εύκολα και από εδώ έχουμε άτοπο.

Άρα δείξαμε ότι γίνεται a=16.

Μένει να δειχτεί ότι δεν γίνεται a<16.

Για να ο επιτύχουμε αυτό, θα βρούμε κατάλληλα b,t ώστε N=b+(b+1)+ \cdots +(b+t-1), με t \neq 2017.

Για a άρτιο, είναι N=2017(a+1008) , και αν 2^s \mid \mid a+1008 \Rightarrow a+1008=2^sm, με m περιττό, έχουμε N=2017 \cdot 2^s \cdot m.

Θέλουμε b+(b+1)+ \cdots +(b+t-1)=2017 \cdot 2^s \cdot m \Rightarrow t(2b+t-1)=2017 \cdot 2^{s+1} \cdot m. Επιλέγουμε t=2^{s+1}, και 2b+t-1=2017m \Rightarrow b=\dfrac{2017m+1-2^{s+1}}{2}.

Αν δείξουμε τώρα ότι b>0, θα έχουμε βρει τα κατάλληλα b,t, που δείχνουν πως δεν γίνεται a<16.

Είναι 2^s \cdot m=a+1008<1024 \Rightarrow s \leqslant 9, άρα b=\dfrac{2017m-2^{s+1}+1}{2} \geqslant \dfrac{2017-2^{10}+1}{2}>0 \Rightarrow b>0.

Για a περιττό, ακολουθούμε την ίδια διαδικασία (s=0).

Έτσι, \textnormal{ \en min} \, a=16.

Edit: Έγιναν κάποιες διορθώσεις. Ευχαριστώ τον Δημήτρη (Demetres).
τελευταία επεξεργασία από Ορέστης Λιγνός σε Τρί Μάιος 29, 2018 9:27 am, έχει επεξεργασθεί 1 φορά συνολικά.


Κερδίζουμε ό,τι τολμούμε!
Άβαταρ μέλους
Demetres
Γενικός Συντονιστής
Δημοσιεύσεις: 8989
Εγγραφή: Δευ Ιαν 19, 2009 5:16 pm
Τοποθεσία: Λεμεσός/Πύλα
Επικοινωνία:

Re: Putnam 2017/B2

#3

Μη αναγνωσμένη δημοσίευση από Demetres » Τρί Μάιος 29, 2018 9:17 am

Σωστά. Όταν όμως δείχνεις την περίπτωση a < 16, το έκανες μόνο για a άρτιο. Ευτυχώς η ίδια λύση δουλεύει και για a περιττό. (Δεν αλλάζει κάτι αν s=0.)

[Υπάρχουν και κάποια τυπογραφικά. Π.χ. εκεί που λες b \neq 1, εννοείς t \neq 1.]


Απάντηση

Επιστροφή σε “Θεωρία Αριθμών - Προχωρημένο Επίπεδο (Seniors)”

Μέλη σε σύνδεση

Μέλη σε αυτήν τη Δ. Συζήτηση: Δεν υπάρχουν εγγεγραμμένα μέλη και 1 επισκέπτης